Evaluating a logarithmic integral using complex analysis

In summary, we discuss the evaluation of the integral $$\int_{0}^{\infty} \frac{\log^2(x)}{x^2 + 1} dx$$using the given contour. We determine that the pole in the contour is $z = i$, and evaluate the residue at this point to be $\frac{-\pi^2}{8i}$. We then use the M-L inequality to show that the integrals along the semicircles $\Gamma_2$ and $\Gamma_4$ converge to $0$ as $R$ and $\epsilon$ approach infinity and $0$ respectively. We also discuss the evaluation of the integral along the $\Gamma_1$ and $\Gamma
  • #1
Amad27
412
1
Hello,

I am evaluating:

$$\int_{0}^{\infty} \frac{\log^2(x)}{x^2 + 1} dx$$

Using the following contour:

View attachment 3742

$R$ is the big radius, $\epsilon$ is small radius (of small circle)

Question before: Which $\log$ branch is this? I asked else they said,

$$-\pi/2 \le arg(z) \le 3\pi/2$$

But in the contour it is: $0 \le \theta \le \pi$ isn't it?

The pole is:

$z = \pm i$, but the one in the contour is $z = i$

$$\text{Res}_{z = i} = \lim_{z = i} \frac{\log^2(z)}{z+i} = \frac{\log^2(i)}{2i}$$

$$\log(z) = \log|z| + iarg(z) \implies \log(i) = \pi(i)/2 \implies \log^2(i) = -\frac{\pi^2}{4}$$

$$\text{Res}_{z = i} = \lim_{z = i} \frac{\log^2(z)}{z+i} = \frac{-\pi^2}{8i}$$

$$\oint_{C} f(z)dz = (2\pi i) \cdot \frac{\pi^2}{8i} = \frac{-\pi^3}{4}$$

Evaluation of $\Gamma_1$:

$$\oint_{\Gamma_1} f(z) dz = \int_{\epsilon}^{R} \frac{\log^2(x)}{x^2 + 1} dx$$

We will take $\epsilon \to 0$ and $R \to \infty$ later down the road.

Evaluation of $\Gamma_2$:

Using $z = Re^{i\theta}$ The denominator $z$'s are there for a reason.

$$\oint_{\Gamma_2} f(z) dz = \int_{0}^{\pi} \frac{\left( \log(R) + i\theta \right )^2 iRe^{i\theta} d\theta}{z)^2 + 1} $$

$$\left | \int_{0}^{\pi} \frac{\left( \log(R) + i\theta \right )^2 iRe^{i\theta} d\theta}{(Re^{i\theta})^2 + 1} \right | \le \int_{0}^{\pi} \frac{(R)\cdot \left | \log(R) + i\theta \right |^2}{\left | (z)^2 + 1 \right |} d\theta$$

$$|\log(R) + i\theta|^2 = \log^2(R) - \theta^2 \le \log^2(R)$$

$$|z^2 + 1| \ge |z|^2 - 1 \implies |z^2 + 1| \ge R^2 - 1$$

Then:

$$\frac{1}{|z^2 + 1|} \le \frac{1}{R^2 - 1}$$

The M-L inequality states:

$$\left | \int \right | \le ML(\Gamma)$$

Estimation lemma - Wikipedia, the free encyclopedia

So $M = \max(|f(z)|)$

$$|f(z)| \le \frac{\log^2(R)}{R^2 - 1}$$

$$L(\Gamma_2) = (1/2)(2\pi R) =\pi R $$

$$\left | \int \right | \le ML(\Gamma)$$

Therefore:

$$\left | \int \right | \le \frac{\log^2(R)\cdot\pi R}{R^2 - 1}$$

$$\lim_{R \to \infty} \left | \int f(z) dz \right | \le \lim_{R \to \infty} \int |f(z) dz| \le \lim_{R \to \infty} \frac{\log^2(R)\cdot\pi R}{R^2 - 1} = 0$$

So $\displaystyle \oint_{\Gamma_2} f(z) dz = 0$

Question: But how do I find the ACTUAL max $M$?? Was that the ABSOLUTE max? The absolute max is the criteria. Help?

Evaluation of $\Gamma_3$:

$$\oint_{\Gamma_3} f(z) dz = \int_{-R}^{-\epsilon} f(x) dx$$

Again, we will take limits, $\epsilon \to 0$ and $R \to \infty$ soon.

Evaluation of $\Gamma_4$:

$$\oint_{\Gamma_4} f(z) dz = \int_{\pi}^{0} \frac{\left( \log(\epsilon) + i\theta \right)^2 i\epsilon e^{i\theta}}{(\epsilon e^{i\theta})^2 + 1} d\theta$$

$$\left | \oint_{\Gamma_4} f(z) dz \right | \le \int_{0}^{\pi} \left | \frac{\left( \log(\epsilon) + i\theta \right)^2 i\epsilon e^{i\theta}}{(\epsilon e^{i\theta})^2 + 1} \right | d\theta $$

$$( \epsilon) \cdot \int_{0}^{\pi} \left | \frac{\left( \log(\epsilon) + i\theta \right)^2}{(\epsilon e^{i\theta})^2 + 1} \right | d\theta $$

$$\left | \left ( \log(\epsilon) + i\theta \right) \right |^2 = \log^2(\epsilon) - \theta^2 \le \log^2(\epsilon) $$

$$|z^2 + 1| \ge \epsilon^2 - 1$$

$$\therefore \frac{1}{|z^2 + 1|} \le \frac{1}{\epsilon^2 - 1}$$

$$|f(z)| \le \frac{\log^2(\epsilon)}{\epsilon^2 - 1} \le M$$

$$L(\Gamma_{4}) = (1/2)(2)(\pi \epsilon) = \pi \epsilon$$

$$\therefore ML(\Gamma) = \frac{\log^2{\epsilon}\pi\epsilon}{\epsilon^2 - 1}$$

$$\left | \oint f(z) dz \right | \le \int |f(z)| dz \le \frac{\log^2(\epsilon)\pi \epsilon}{\epsilon^2 - 1}$$

$$\lim_{epsilon \to 0} \left | \oint f(z) dz \right | \le \lim_{epsilon \to 0} \int |f(z)| dz \le \lim_{epsilon \to 0} \frac{\log^2(\epsilon)\pi \epsilon}{\epsilon^2 - 1}$$So:

$$\oint_{C} = \int_{\epsilon}^{R} \frac{\log^2(x)}{x^2 + 1} dx + \int_{0}^{\pi} \frac{\left( \log(R) + i\theta \right )^2 iRe^{i\theta} d\theta}{z)^2 + 1} + \int_{-R}^{-\epsilon} \frac{\log^2(x)}{x^2 + 1} dx + \int_{\pi}^{0} \frac{\left( \log(\epsilon) + i\theta \right)^2 i\epsilon e^{i\theta}}{(\epsilon e^{i\theta})^2 + 1} d\theta$$

$$\lim{R \to \infty}_{\epsilon \to 0} \oint_{C} = \int_{0}^{\infty} \frac{\log^2(x)}{x^2 + 1} dx + \int_{0}^{\infty} \frac{\log^2(-x)}{x^2 + 1} dx = \frac{-\pi^3}{4}$$

Therefore,

$$ \int_{0}^{\infty} \frac{\log^2(x)}{x^2 + 1} dx + \int_{0}^{\infty} \frac{\log^2(-x)}{x^2 + 1} dx = \frac{-\pi^3}{4}$$

Then we use $\log(z) = \log(|z|) + iarg(z)$ to get the required integral.

Can you just check the proofs for the integrals converging to $0$?
 

Attachments

  • Log contour.png
    Log contour.png
    3.5 KB · Views: 60
Last edited:
Physics news on Phys.org
  • #2
The fact that the integral is negative should make you think that there must be something wrong somewhere... isn't it?... take into account that, solving the integral with the procedure illustrated in...

http://mathhelpboards.com/math-notes-49/integrals-natural-logarithm-5286.html

... the result should be...

$\displaystyle \int_{0}^{\infty} \frac{\ln ^{2} x}{1 + x^{2}} \ d x = \int_{0}^{1} \frac{\ln ^{2} x}{1 + x^{2}} \ d x + \int_{1}^{\infty} \frac{\ln ^{2} x}{1 + x^{2}}\ d x = \int_{1}^{\infty} \frac{\ln ^{2} x}{1 + x^{2}}\ d x + \int_{1}^{\infty} \frac{\ln ^{2} x}{1 + x^{2}}\ d x = $

$\displaystyle = 2\ \int_{1}^{\infty} \frac{\ln ^{2} x}{1 + x^{2}}\ d x = 4\ \sum_{k=0}^{\infty} \frac{(-1)^{k}}{(2 k + 1)^{3}} = \frac{\pi ^{3}}{8}$

http://d16cgiik7nzsna.cloudfront.net/82/e7/i98953090._szw380h285_.jpgMerry Christmas from Serbia

$\chi$ $\sigma$
 
  • #3
Olok said:
Evaluation of $\Gamma_3$:

$$\oint_{\Gamma_3} f(z) dz = \int_{-R}^{-\epsilon} f(x) dx$$

Does that imply that

$$\frac{\log^2(x)}{1+x^2} = \frac{\log^2(z)}{1+z^2}$$

On the negative x-axis ?
 
  • #4
chisigma said:
The fact that the integral is negative should make you think that there must be something wrong somewhere... isn't it?... take into account that, solving the integral with the procedure illustrated in...

http://mathhelpboards.com/math-notes-49/integrals-natural-logarithm-5286.html

... the result should be...

$\displaystyle \int_{0}^{\infty} \frac{\ln ^{2} x}{1 + x^{2}} \ d x = \int_{0}^{1} \frac{\ln ^{2} x}{1 + x^{2}} \ d x + \int_{1}^{\infty} \frac{\ln ^{2} x}{1 + x^{2}}\ d x = \int_{1}^{\infty} \frac{\ln ^{2} x}{1 + x^{2}}\ d x + \int_{1}^{\infty} \frac{\ln ^{2} x}{1 + x^{2}}\ d x = $

$\displaystyle = 2\ \int_{1}^{\infty} \frac{\ln ^{2} x}{1 + x^{2}}\ d x = 4\ \sum_{k=0}^{\infty} \frac{(-1)^{k}}{(2 k + 1)^{3}} = \frac{\pi ^{3}}{8}$

Merry Christmas from Serbia

$\chi$ $\sigma$

$$I = \int_{0}^{\infty} \frac{\log^2(x)}{x^2 + 1} \,dx + \int_{0}^{\infty} \frac{\log^2(-x)}{x^2 + 1} \,dx $$

$$\log(-x) = \log(x) + i\theta$$

Whatever theta is in $0 \le \theta \le \pi$$$I = \int_{0}^{\infty} \frac{\log^2(x)}{x^2 + 1} \,dx + \int_{0}^{\infty} \frac{\left( \log(x) + i\theta \right)^2}{x^2 + 1} \,dx$$

$$I = \int_{0}^{\infty} \frac{\log^2(x)}{x^2 + 1} \,dx + \int_{0}^{\infty} \frac{\log^2(x) + 2i\theta\log(x) - \theta^2}{x^2 + 1} \,dx$$

Because theta is $\pi$ for the negative x-axis and $0$ for positive x-axis.

$$I = 2\int_{0}^{\infty} \frac{\log^2(x)}{x^2 + 1} \,dx + \int_{0}^{\infty} \frac{2i\pi\log(x) - \pi^2}{x^2 + 1} \,dx$$

$$I = 2\int_{0}^{\infty} \frac{\log^2(x)}{x^2 + 1} \,dx + 0 - \pi^2 \int_{0}^{\infty} \frac{1}{x^2 + 1} dx = 2\int_{0}^{\infty} \frac{\log^2(x)}{x^2 + 1} \,dx - (\pi^3)/2 = -\pi^3/4$$

$$2I = \frac{\pi^3}{4}$$

$$I = \frac{\pi^3}{8}$$

Now that that is done, I still need a way to prove:

$$\int_{0}^{\infty} \frac{2\pi\log(x)}{x^2 + 1} dx = 0$$

With complex analysis.

We could use the substitution $t = 1/x$ then prove that $-I = I$ which implies $I=0$ but that isn't rigorous at all. The goal is

Prove:

$$\int_{0}^{\infty} \frac{\log(x)}{x^2 + 1} dx = 0$$

I don't know, well, we'll see or we can always justify $x = 1/t$ so I suppose its fine.

Also, just a question,

How can we use contour integration with integrals from $0 \to 1$

My best guess would be to use the unit circle??

Thanks, and Merry Christmas from Turkmenistan.

@Zaid, it does imply the negative x-axis.
 
  • #5
Olok said:
$$I = \int_{0}^{\infty} \frac{\log^2(x)}{x^2 + 1} \,dx + \int_{0}^{\infty} \frac{\log^2(-x)}{x^2 + 1} \,dx $$

I would rewrite it as

$$\int_{-\infty}^0 \frac{(\ln |x|+i\pi)^2}{x^2+1}\,dx = \int_{0}^\infty \frac{(\ln (x)+i\pi)^2}{x^2+1}\,dx $$

For the other integral

$$I=\int^\infty_0 \frac{\log(x)}{x^2+1}\,dx$$

You can use the same method by integrating

$$f(z)= \frac{\log(z)}{1+z^2}$$

or , simply prove that $I=-I$.
 
  • #6
ZaidAlyafey said:
I would rewrite it as

$$\int_{-\infty}^0 \frac{(\ln |x|+i\pi)^2}{x^2+1}\,dx = \int_{0}^\infty \frac{(\ln (x)+i\pi)^2}{x^2+1}\,dx $$

For the other integral

$$I=\int^\infty_0 \frac{\log(x)}{x^2+1}\,dx$$

You can use the same method by integrating

$$f(z)= \frac{\log(z)}{1+z^2}$$

or , simply prove that $I=-I$.

It is actually very complicated from research, Ill work on it tomorrow and post updates.
 

Related to Evaluating a logarithmic integral using complex analysis

1. What is a logarithmic integral?

A logarithmic integral is a mathematical function that represents the integral of the natural logarithm function. It is often denoted as Li(x) and can be used to evaluate certain types of integrals that involve logarithmic terms.

2. How is complex analysis used to evaluate a logarithmic integral?

Complex analysis is a branch of mathematics that deals with functions of complex numbers. It can be used to evaluate logarithmic integrals by converting them into complex integrals, which can then be solved using techniques such as contour integration.

3. What are the benefits of using complex analysis to evaluate a logarithmic integral?

Using complex analysis to evaluate a logarithmic integral can often simplify the integration process and yield more accurate results. It also allows for the use of powerful techniques and concepts, such as Cauchy's integral theorem and residue calculus.

4. What types of integrals can be evaluated using complex analysis?

Complex analysis can be used to evaluate a wide range of integrals, including those involving logarithmic, trigonometric, and exponential functions. It is particularly useful for evaluating integrals that are difficult to solve using other methods.

5. Are there any limitations to using complex analysis for evaluating logarithmic integrals?

While complex analysis can be a powerful tool for evaluating logarithmic integrals, it does have its limitations. It may not be suitable for all types of integrals, and it requires a good understanding of complex numbers and analytic functions. Additionally, it may not always provide a closed form solution for the integral.

Similar threads

Replies
4
Views
774
  • Topology and Analysis
Replies
2
Views
662
  • Topology and Analysis
Replies
2
Views
1K
  • Topology and Analysis
Replies
14
Views
497
Replies
4
Views
1K
  • Topology and Analysis
Replies
4
Views
305
  • Topology and Analysis
Replies
1
Views
430
Replies
4
Views
393
  • Topology and Analysis
Replies
1
Views
1K
Replies
6
Views
883
Back
Top